Appendix A — Revision Guide
A.1 Recommended revision strategy
Make sure you are very comfortable with the Definitions, Statements, Proofs, and Examples contained in this Revision Guide.
Make sure you are comfortable with the Tutorial and Homework questions.
Make sure you are comfortable with the 2023/24 Exam Paper questions.
Make sure you are comfortable with the Checklist in Section A.2.
A.2 Checklist
Overall, you should be comfortable with the following topics/taks:
Preliminaries
- Prove that \(\sqrt{p} \notin \mathbb{Q}\) for \(p\) a prime number.
Complex Numbers
- Sum, multiplication and division of complex numbers
- Computing the complex conjugate
- Computing the inverse of a complex number
- Find modulus and argument of a complex number
- Compute Cartesian, Trigonometric and Exponential form of a complex number
- Complex exponential and its properties
- Computing powers of complex numbers
- Solving degree 2 polynomial equations in \(\mathbb{C}\)
- Long division of polynomials
- Solving higher degree polynomial equations in \(\mathbb{C}\)
- Finding the roots of unity
- Finding the n-th roots of a complex number
A.3 Preliminaries
Theorem 1
Proof
Aassume by contradiction that \[ \sqrt{2} \in \mathbb{Q}\,. \tag{A.1}\]
Therefore, there exists \(q \in \mathbb{Q}\) such that \[ q = \sqrt{2} \, . \tag{A.2}\]
Since \(q \in \mathbb{Q}\), by definition we have \[ q = \frac{m}{n} \] for some \(m,n \in \mathbb{N}\) with \(n \neq 0\).
Recalling (A.2), we then have \[ \frac{m}{n} = \sqrt{2} \,. \]
We can square the above equation to get \[ \frac{m^2}{n^2} = 2 \,. \tag{A.3}\]
Withouth loss of generality, we can assume that \(m\) and \(n\) have no common factors.
Equation (A.3) implies \[ m^2 = 2 n^2 \,. \tag{A.4}\] Therefore the integer \(m^2\) is an even number.
Since \(m^2\) is an even number, it follows that also \(m\) is an even number. Then there exists \(p \in \mathbb{N}\) such that \[ m = 2p \,. \tag{A.5}\]
If we substitute (A.5) in (A.4) we get \[ m^2 = 2n^2 \, \implies \, (2p)^2 = 2n^2 \, \implies \, 4 p^2 = 2 n^2 \] Dividing both terms by \(2\), we obtain \[ n^2 = 2p^2\,. \tag{A.6}\]
We now make a series of observations:
- Equation (A.6) says that \(n^2\) is even.
- The same argument in Step 7 guarantees that also \(n\) is even.
- We have already seen that \(m\) is even.
- Therefore \(n\) and \(m\) are both even.
- Hence \(n\) and \(m\) have \(2\) as common factor.
- But Step 5 says that \(n\) and \(m\) have no common factors.
- Contradiction
Our reasoning has run into a contradiction, stemming from assumption (A.1). Therefore (A.1) is FALSE, and so \[ \sqrt{2} \notin \mathbb{Q}\, \] ending the proof.
A.3.1 Set Theory
Definition 2
Proposition 3
Definition 4
Let \(\Omega\) be a set, and \(A_n \subseteq \Omega\) a family of subsets, where \(n \in \mathbb{N}\).
The infinte union of the \(A_n\) is the set \[ \bigcup_{n \in \mathbb{N}} A_n := \{ x \in \Omega \, \colon \, x \in A_n \,\, \text{ for at least one } \,\, n \in \mathbb{N}\} \,. \]
The infinte intersection of the \(A_n\) is the set \[ \bigcap_{n \in \mathbb{N}} A_n := \{ x \in \Omega \, \colon \, x \in A_n \,\, \text{ for all } \,\, n \in \mathbb{N}\} \,. \]
Example 5
Question. Define \(\Omega:=\mathbb{N}\) and a family \(A_n\) by \[ A_n = \{ n, n+1, n+2, n+3, \ldots \} \,, \quad n \in \mathbb{N}\,. \]
Prove that \[ \bigcup_{n \in \mathbb{N}} A_n = \mathbb{N}\,. \tag{A.7}\]
Prove that \[ \bigcap_{n \in \mathbb{N}} A_n = \emptyset \,. \tag{A.8}\]
Solution.
Assume that \(m \in \cup_n A_n\). Then \(m \in A_n\) for at least one \(n \in \mathbb{N}\). Since \(A_n \subseteq \mathbb{N}\), we conclude that \(m \in \mathbb{N}\). This shows \[ \bigcup_{n \in \mathbb{N}} A_n \subseteq \mathbb{N}\,. \] Conversely, suppose that \(m \in \mathbb{N}\). By definition \(m \in A_m\). Hence there exists at least one index \(n\), \(n=m\) in this case, such that \(m \in A_n\). Then by definition \(m \in \cup_{n \in \mathbb{N}} A_n\), showing that \[ \mathbb{N}\subseteq \bigcup_{n \in \mathbb{N}} A_n \,. \] This proves (A.7).
Suppose that (A.8) is false, i.e., \[ \bigcap_{n \in \mathbb{N}} A_n \neq \emptyset \,. \] This means there exists some \(m \in \mathbb{N}\) such that \(m \in \cap_{n \in \mathbb{N}} A_n\). Hence, by definition, \(m \in A_n\) for all \(n \in \mathbb{N}\). However \(m \notin A_{m+1}\), yielding a contradiction. Thus (A.8) holds.
Definition 6
Example 7
Question. Suppose \(A, B \subseteq \Omega\). Prove that \[ A \subseteq B \iff B^c \subseteq A^c \,. \]
Solution. Let us prove the above claim:
First implication \(\implies\):
Suppose that \(A \subseteq B\). We need to show that \(B^c \subseteq A^c\). Hence, assume \(x \in B^c\). By definition this means that \(x \notin B\). Now notice that we cannot have that \(x \in A\). Indeed, assume \(x \in A\). By assumption we have \(A \subseteq B\), hence \(x \in B\). But we had assumed \(x \in B\), contradiction. Therefore it must be that \(x \notin A\). Thus \(B^c \subseteq A^c\).Second implication \(\impliedby\): Note that, for any set, \[ (A^c)^c = A \,. \] Hence, by the first implication, \[ B^c \subseteq A^c \, \implies \, (A^c)^c \subseteq (B^c)^c \, \implies \, A \subseteq B \,. \]
Proposition 8: De Morgan’s Laws
Definition 9
Example 10
Solution. \(\mathcal{P}(\Omega)\) has \(2^3 = 8\), and \[\begin{align} \mathcal{P}(\Omega) = \{ \emptyset, & \, \{x\} , \, \{y\} , \, \{z\} , \{x,y\} \\ & \{x,z\}, \, \{y,z\} , \, \{x,y,z\} \} \,. \end{align}\]
Definition 11
A.3.2 Relations
Definition 12
Definition 13: Equivalence relation
Reflexive: For each \(x \in A\) one has \[ (x,x) \in R \,, \]
Symmetric: We have \[ (x,y) \in R \implies (y,x) \in R \]
Transitive: We have \[ (x,y) \in R \,, \,\, (y,z) \in R \implies (x,z) \in R \]
If \((x,y) \in R\) we write \[ x \sim y \] and we say that \(x\) and \(y\) are equivalent.
Definition 14: Equivalence classes
Proposition 15
Let \(\sim\) be an equivalence relation on \(A\). Then
For each \(x \in A\) we have \[ [x] \neq \emptyset \]
For all \(x,y \in A\) it holds \[ x \sim y \quad \iff \quad [x] = [y] \,. \]
Example 16: Equality is an equivalence relation
Question. The equality defines a binary relation on \(\mathbb{Q}\times \mathbb{Q}\), via \[ R:=\{ (x,y) \in \mathbb{Q}\times \mathbb{Q}\, \colon \, x=y \} \,. \]
- Prove that \(R\) is an equivalence relation.
- Prove that \([x] = \{x\}\) and compute \(\mathbb{Q}/ R\).
Solution.
We need to check that \(R\) satisfies the 3 properties of an equivalence relation:
Reflexive: It holds, since \(x=x\) for all \(x \in \mathbb{Q}\),
Symmetric: Again \(x = y\) if and only if \(y = x\),
Transitive: If \(x = y\) and \(y = z\) then \(x = z\).
Therefore, \(R\) is an equivalence relation.
The class of equivalence of \(x \in \mathbb{Q}\) is given by \[ [x] = \{ x \}\,, \] that is, this relation is quite trivial, given that each element of \(\mathbb{Q}\) can only be related to itself. The quotient space is then \[ \mathbb{Q}/ R = \{ [x] \, \colon \,x \in \mathbb{Q}\} = \{ \{ x\} \, \colon \,x \in \mathbb{Q}\} \,. \]
Example 17
Question. Let \(R\) be the binary relation on the set \(\mathbb{Q}\) of rational numbers defined by \[ x \sim y \iff x-y \in \mathbb{Z}\,. \]
- Prove that \(R\) is an equivalence relation on \(\mathbb{Q}\).
- Compute \([x]\) for each \(x \in \mathbb{Q}\).
- Compute \(\mathbb{Q}/R\).
Solution.
We have:
Reflexive: Let \(x \in \mathbb{Q}\). Then \(x-x = 0\) and \(0 \in \mathbb{Z}\). Thus \(x \sim x\).
Symmetric: If \(x \sim y\) then \(x-y \in \mathbb{Z}\). But then also
\[ -(x-y) = y - x \in \mathbb{Z} \] and so \(y \sim x\).Transitive: Suppose \(x \sim y\) and \(y \sim z\). Then \[ x - y \in \mathbb{Z}\, \text{ and } \, y - z \in \mathbb{Z}\,. \] Thus, we have \[ x-z = (x-y) + (y-z) \in \mathbb{Z} \] showing that \(x \sim z\).
Thus, we have shown that \(R\) is an equivalence relation on \(\mathbb{Q}\).
Note that \[ y -x \in \mathbb{Z} \] is equivalent to \[ \exists \, n \in \mathbb{Z}\, \text{ s.t. } \, y - x = n \] which again is equivalent to \[ \exists \, n \in \mathbb{Z}\, \text{ s.t. } \, y = x + n \,. \] In summary we have \[ x \sim y \quad \iff \quad \exists \, n \in \mathbb{Z}\, \text{ s.t. } \, y = x + n \,. \] Therefore the equivalence classes with respect to \(\sim\) are \[ [x] = \{ x + n \ \colon \ n \in \mathbb{Z}\} \,. \] Each equivalence class has exactly one element in \([0,1) \cap \mathbb{Q}\), meaning that: \[ \forall x \in \mathbb{Q}\,, \,\, \exists! \, q \in \mathbb{Q}\, \text{ s.t. } \, 0 \leq q < 1 \, \text{ and } \, q \in [x]\,. \tag{A.9}\] Indeed: take \(x \in \mathbb{Q}\) arbitrary. Then \(x \in [n,n+1)\) for some \(n \in \mathbb{Z}\). Setting \(q:=x-n\) we obtain that \[ x = q + n \,, \qquad q \in [0,1) \,, \] proving (A.9). In particular (A.9) implies that for each \(x \in \mathbb{Q}\) there exists \(q \in [0,1) \cap \mathbb{Q}\) such that \[ [x] = [q] \,. \]
From Point 2 we conclude that \[ \mathbb{Q}/ R = \{ [x] \, \colon \,x \in \mathbb{Q}\} = \{ q \in \mathbb{Q}\, \colon \,0 \leq q <1 \} \,. \]
Definition 18: Partial order
A binary relation \(R\) on \(A\) is called a partial order if it satisfies the following properties:
Reflexive: For each \(x \in A\) one has \[ (x,x) \in R \,, \]
Antisymmetric: We have \[ (x,y) \in R \, \text{ and } \, (y,x) \in R \implies x = y \]
Transitive: We have \[ (x,y) \in R \,, \,\, (y,z) \in R \implies (x,z) \in R \]
Definition 19: Total order
A binary relation \(R\) on \(A\) is called a total order relation if it satisfies the following properties:
- Partial order: \(R\) is a partial order on \(A\).
- Total: For each \(x,y \in A\) we have \[ (x, y) \in R \,\, \mbox{ or } \,\, (y,x) \in R \,. \]
Example 20: Set inclusion is a partial order but not total order
Question. Let \(\Omega\) be a non-empty set and consider its power set \[ \mathcal{P}(\Omega) = \{ A \, \colon \,A \subseteq \Omega \}\,. \] The inclusion defines binary relation on \(\mathcal{P}(\Omega) \times \mathcal{P}(\Omega)\), via \[ R:=\{ (A,B) \in \mathcal{P}(\Omega) \times \mathcal{P}(\Omega) \, \colon \, A \subseteq B \} \,. \]
- Prove that \(R\) is an order relation.
- Prove that \(R\) is not a total order.
Solution.
- Check that \(R\) is a partial order relation on \(\mathcal{P}(\Omega)\):
- Reflexive: It holds, since \(A \subseteq A\) for all \(A \in \mathcal{P}(\Omega)\).
- Antisymmetric: If \(A \subseteq B\) and \(B \subseteq A\), then \(A=B\).
- Transitive: If \(A \subseteq B\) and \(B \subseteq C\), then, by definition of inclusion, \(A \subseteq C\).
- In general, \(R\) is not a total order. For example consider \[ \Omega = \{x, y\}\,. \] Thus \[ \mathcal{P}(\Omega) = \{ \emptyset , \, \{x\} , \, \{y\} , \, \{x,y\} \}\,. \] If we pick \(A=\{x\}\) and \(B=\{y\}\) then \(A \cap B = \emptyset\), meaning that \[ A \not\subseteq B \,, \quad B \not\subseteq A \,. \] This shows \(R\) is not a total order.
Example 21: Inequality is a total order
Solution. We need to check that:
Reflexive: It holds, since \(x \leq x\) for all \(x \in \mathbb{Q}\),
Antisymmetric: If \(x \leq y\) and \(y \leq x\) then \(x = y\).
Transitive: If \(x \leq y\) and \(y \leq z\) then \(x \leq z\).
Finally, we halso have that \(R\) is a total order on \(\mathbb{Q}\), since for all \(x,y \in \mathbb{Q}\) we have \[ x \leq y \,\, \text{ or } \,\, y \leq x \,. \]
A.3.3 Absolute value
Definition 22: Absolute value
Proposition 23
For all \(x \in \mathbb{R}\) they hold:
- \(|x| \geq 0\).
- \(|x| = 0\) if and only if \(x = 0\).
- \(|x| = |-x|\).
Lemma 24
Corollary 25
Theorem 26: Triangle inequality
Proposition 27
A.3.4 Induction
Definition 28: Principle of Inducion
- \(\alpha(1)\) is true, and
- Whenever \(\alpha(n)\) is true, then \(\alpha(n+1)\) is true.
Then \(\alpha(n)\) is true for all \(n \in \mathbb{N}\).
Example 29: Formula for summing first \(n\) natural numbers
Solution. Define \[ S(n) = 1 + 2 + \ldots + n \,. \] This way the formula at (A.12) is equivalent to \[ S(n) = \frac{n(n+1)}{2} \,, \quad \forall \, n \in \mathbb{N}\,. \]
- It is immediate to check that (A.12) holds for \(n=1\).
- Suppose (A.12) holds for \(n = k\). Then \[\begin{align} S(k+1) & = 1 + \ldots + k + (k+1) \\ & = S(k) + (k+1) \\ & = \frac{k(k+1)}{2} + (k+1) \\ & = \frac{ k(k+1) + 2(k+1) }{2} \\ & = \frac{(k+1)(k+2)}{2} \end{align}\] where in the first equality we used that (A.12) holds for \(n=k\). We have proven that \[ S(k+1) = \frac{(k+1)(k+2)}{2}\,. \] The RHS in the above expression is exactly the RHS of (A.12) computed at \(n = k+1\). Therefore, we have shown that formula (A.12) holds for \(n = k+1\).
By the Principle of Induction, we conclude that (A.12) holds for all \(n \in \mathbb{N}\).
Example 30: Bernoulli’s inequality
Solution. Let \(x \in \mathbb{R}, x>-1\). We prove the statement by induction:
Base case: (A.33) holds with equality when \(n=1\).
Induction hypothesis: Let \(k \in \mathbb{N}\) and suppose that (A.33) holds for \(n=k\), i.e., \[ (1+x)^{k} \geq 1+k x \,. \] Then \[\begin{align*} (1+x)^{k+1} & = (1+x)^{k}(1+x) \\ & \geq(1+k x)(1+x) \\ & =1+k x+x+k x^{2} \\ & \geq 1+(k+1) x \,, \end{align*}\] where we used that \(kx^2 \geq 0\). Then (A.33) holds for \(n=k+1\).
By induction we conclude (A.33).
A.4 Real Numbers
A.4.1 Fields
Definition 31: Binary operation
Definition 32
Let \(K\) be a set and \(\circ \ \colon K \times K \to K\) be a binary operation on \(K\). We say that:
- \(\circ\) is commutative if \[ x \circ y = y \circ x \,, \quad \, \forall \, x,y \in K \]
- \(\circ\) is associative if \[ (x \circ y) \circ z = x \circ (y \circ z) \,, \quad \, \forall \, x,y,z \in K \]
- An element \(e \in K\) is called neutral element of \(\circ\) if \[ x \circ e = e \circ x = x \,, \quad \, \forall \, x \in K \]
- Let \(e\) be a neutral element of \(\circ\) and let \(x \in K\). An element \(y \in K\) is called an inverse of \(x\) with respect to \(\circ\) if \[ x \circ y = y \circ x = e \,. \]
Example 33
Question. Let \(K=\{0,1\}\) be a set with binary operation \(\circ\) defined by the table \[ \begin{array}{c|cc} \circ & 0 & 1 \\ \hline 0 & 1 & 1 \\ 1 & 0 & 0 \\ \end{array} \]
Is \(\circ\) commutative? Justify your answer.
Is \(\circ\) associative? Justify your answer.
Solution.
We have \[ 0 \circ 1 = 1 \, , \quad 1 \circ 0 = 0 \] and therefore \[ 0 \circ 1 \neq 1 \circ 0 \,. \] showing that \(\circ\) is not commutative.
We have \[ (0 \circ 1) \circ 1 = 1 \circ 1 = 0 \,, \] while \[ 0 \circ (1 \circ 1) = 0 \circ 0 = 1 \,, \] so that \[ (0 \circ 1) \circ 1 \neq 0 \circ (1 \circ 1)\,. \] Thus, \(\circ\) is not associative.
Definition 34: Field
Let \(K\) be a set with binary operations of addition \[ +\ \colon K \times K \to K \,, \quad (x,y) \mapsto x + y \] and multiplication \[ \cdot\ \colon K \times K \to K \,, \quad (x,y) \mapsto x \cdot y = xy \,. \] We call the triple \((K, + , \cdot)\) a field if:
- The addition \(+\) satisfies: \(\,\forall \, x,y,z \in K\)
- (A1) Commutativity and Associativity: \[ x+y = y+x \] \[ (x+y)+z = x+(y+z) \]
- (A2) Additive Identity: There exists a neutral element in \(K\) for \(+\), which we call \(0\). It holds: \[ x + 0 = 0 + x = x \]
- (A3) Additive Inverse: There exists an inverse of \(x\) with respect to \(+\). We call this element the additive inverse of \(x\) and denote it by \(-x\). It holds \[ x + (-x) = (-x) + x = 0 \]
- The multiplication \(\cdot\) satisifes: \(\,\forall \, x,y,z \in K\)
- (M1) Commutativity and Associativity: \[ x \cdot y = y \cdot x \] \[ (x \cdot y) \cdot z = x \cdot (y \cdot z) \]
- (M2) Multiplicative Identity: There exists a neutral element in \(K\) for \(\cdot\), which we call \(1\). It holds: \[ x \cdot 1 = 1 \cdot x = x \]
- (M3) Multiplicative Inverse: If \(x \neq 0\) there exists an inverse of \(x\) with respect to \(\cdot\). We call this element the multiplicative inverse of \(x\) and denote it by \(x^{-1}\). It holds \[ x \cdot x^{-1} = x^{-1} \cdot x = 1 \]
- The operations \(+\) and \(\cdot\) are related by
- (AM) Distributive Property: \(\,\forall \, x,y,z \in K\) \[ x \cdot (y + z) = (x \cdot y) + (y \cdot z) \,. \]
Theorem 35
Consider the sets \(\mathbb{N}\), \(\mathbb{Z}\), \(\mathbb{Q}\) with the usual operations \(+\) and \(\cdot\). We have:
\((\mathbb{N}, + , \cdot)\) is not a field.
\((\mathbb{Z}, + , \cdot)\) is not a field.
\((\mathbb{Q}, + , \cdot)\) is a field.
Theorem 36
Proposition 37: Uniqueness of neutral elements and inverses
Let \((K,+,\cdot)\) be a field. Then
- There is a unique element in \(K\) with the property of \(0\).
- There is a unique element in \(K\) with the property of \(1\).
- For all \(x \in K\) there is a unique additive inverse \(-x\).
- For all \(x \in K\), \(x \neq 0\), there is a unique multiplicative inverse \(x^{-1}\).
Proof
- Suppose that \(0 \in K\) and \(\widetilde{0} \in K\) are both neutral element of \(+\), that is, they both satisfy (A2). Then \[ 0 + \widetilde{0} = 0 \] since \(\widetilde{0}\) is a neutral element for \(+\). Moreover \[ \widetilde{0} + 0 = \widetilde{0} \] since \(0\) is a neutral element for \(+\). By commutativity of \(+\), see property (A1), we have \[ 0 = 0 + \widetilde{0} = \widetilde{0} + 0 = \widetilde{0} \,, \] showing that \(0 = \widetilde{0}\). Hence the neutral element for \(+\) is unique.
- Exercise.
- Let \(x \in K\) and suppose that \(y, \widetilde{y} \in K\) are both additive inverses of \(x\), that is, they both satisfy (A3). Therefore \[ x + y = 0 \] since \(y\) is an additive inverse of \(x\) and \[ x + \widetilde{y} = 0 \] since \(\widetilde{y}\) is an additive inverse of \(x\). Therefore we can use commutativity and associativity and of \(+\), see property (A1), and the fact that \(0\) is the neutral element of \(+\), to infer \[\begin{align*} y & = y + 0 = y + (x + \widetilde{y}) \\ & = (y + x) + \widetilde{y} = (x + y) + \widetilde{y} \\ & = 0 + \widetilde{y} = \widetilde{y} \,, \end{align*}\] concluding that \(y = \widetilde{y}\). Thus there is a unique additive inverse of \(x\), and \[ y = \widetilde{y} = -x \,, \] with \(-x\) the element from property (A3).
- Exercise.
Definition 38
Let \(K\) be a set with binary operations \(+\) and \(\cdot\), and with an order relation \(\leq\). We call \((K,+,\cdot,\leq)\) an ordered field if:
\((K,+,\cdot)\) is a field
There \(\leq\) is of total order on \(K\): \(\, \forall \, x, y, z \in K\)
- (O1) Reflexivity: \[ x \leq x \]
- (O2) Antisymmetry: \[ x \leq y \, \mbox{ and } \, y \leq x \,\, \implies \,\, x = y \]
- (O3) Transitivity: \[ x \leq y \,\, \mbox{ and } \,\, y \leq z \,\, \implies \,\, x = z \]
- (O4) Total order:
\[ x \leq y \,\, \mbox{ or } \,\, y \leq x \]
The operations \(+\) and \(\cdot\), and the total order \(\leq\), are related by the following properties: \(\, \forall x, y, z \in K\)
- (AM) Distributive: Relates addition and multiplication via \[ x \cdot (y + z) = x \cdot y + x \cdot z \]
- (AO) Relates addition and order with the requirement: \[ x \leq y \,\, \implies \,\, x + z \leq y + z \]
- (MO) Relates multiplication and order with the requirement: \[ x \geq 0, \, y \geq 0 \,\, \implies \,\, x \cdot y \geq 0 \]
Theorem 39
A.4.2 Supremum and infimum
Definition 40: Upper bound, bounded above, supremum, maximum
Let \(A \subseteq K\):
We say that \(b \in K\) is an upper bound for \(A\) if \[ a \leq b \,, \quad \forall \, a \in A \,. \]
We say that \(A\) is bounded above if there exists and upper bound \(b \in K\) for \(A\).
We say that \(s \in K\) is the least upper bound or supremum of \(A\) if:
- \(s\) is an upper bound for \(A\),
- \(s\) is the smallest upper bound of \(A\), that is, \[ \mbox{If } \, b \in K \, \mbox{ is upper bound for } \, A \, \mbox{ then } \, s \leq b \,. \] If it exists, the supremum is denoted by \[ s = \sup \ A \,. \]
Let \(A \subseteq K\). We say that \(M \in K\) is the maximum of \(A\) if: \[ M \in A \,\, \mbox{ and } \,\, a \leq M \,, \, \forall a \in A \,. \] If it exists, we denote the maximum by \[ M = \max A \,. \]
Remark 41
Proposition 42: Relationship between Max and Sup
Definition 43: Upper bound, bounded below, infimum, minimum
Let \(A \subseteq K\):
We say that \(l \in K\) is a lower bound for \(A\) if \[ l \leq a \,, \quad \forall \, a \in A \,. \]
We say that \(A\) is bounded below if there exists a lower bound \(l \in K\) for \(A\).
We say that \(i \in K\) is the greatest lower bound or infimum of \(A\) if:
- \(i\) is a lower bound for \(A\),
- \(i\) is the largest lower bound of \(A\), that is, \[ \mbox{If } \, l \in K \, \mbox{ is a lower bound for } \, A \, \mbox{ then } \, l \leq i \,. \] If it exists, the infimum is denoted by \[ i = \inf A \,. \]
We say that \(m \in K\) is the minimum of \(A\) if: \[ m \in A \,\, \mbox{ and } \,\, m \leq a \,, \, \forall a \in A \,. \] If it exists, we denote the minimum by \[ m = \min A \,. \]
Proposition 44
Proposition 45
Proposition 46: Relationship between sup and inf
Let \(A \subseteq K\). Define \[ - A := \{ - a \, \colon \,a \in A \} \,. \] They hold
- If \(\sup A\) exists, then \(\inf A\) exists and \[ \inf(-A) = - \sup A \,. \]
- If \(\inf A\) exists, then \(\sup A\) exists and \[ \sup(-A) = - \inf A \,. \]
A.4.3 Axioms of Real Numbers
Definition 47: Completeness
Let \((K,+,\cdot,\leq)\) be an ordered field. We say that \(K\) is complete if it holds the property:
- (AC) For every \(A \subseteq K\) non-empty and bounded above \[ \sup A \in K \,. \]
Theorem 48
- \(A\) is non-empty,
- \(A\) is bounded above,
- \(\sup A\) does not exist in \(\mathbb{Q}\).
One of such sets is, for example, \[ A = \{ q \in \mathbb{Q}\, \colon \,q \geq 0 \,, \,\, q^2 < 2 \} \,. \]
Proposition 49
Definition 50: System of Real Numbers \(\mathbb{R}\)
A system of Real Numbers is a set \(\mathbb{R}\) with two operations \(+\) and \(\cdot\), and a total order relation \(\leq\), such that
\((\mathbb{R},+,\cdot, \leq)\) is an ordered field
\(\mathbb{R}\) sastisfies the Axiom of Completeness
A.4.4 Inductive sets
Definition 51: Inductive set
Let \(S \subseteq \mathbb{R}\). We say that \(S\) is an inductive set if they are satisfied:
- \(1 \in S\),
- If \(x \in S\), then \((x + 1) \in S\).
Example 52
Question. Prove the following:
\(\mathbb{R}\) is an inductive set.
The set \(A=\{0,1\}\) is not an inductive set.
Solution.
We have that \(1 \in \mathbb{R}\) by axiom (M2). Moreover \((x + 1) \in \mathbb{R}\) for every \(x \in \mathbb{R}\), by definition of sum \(+\).
We have \(1 \in A\), but \((1 + 1) \notin A\), since \(1 + 1 \neq 0\).
Proposition 53
Definition 54: Set of Natural Numbers
Proposition 55: \({\mathbb{N}}_{\mathbb{R}}\) is the smallest inductive subset of \(\mathbb{R}\)
Theorem 56
A.5 Properties of \(\mathbb{R}\)
Theorem 57: Archimedean Property
Let \(x \in \mathbb{R}\) be given. Then:
There exists \(n \in \mathbb{N}\) such that \[ n > x \,. \]
Suppose in addition that \(x>0\). There exists \(n \in \mathbb{N}\) such that \[ \frac1n < x \,. \]
Theorem 58: Archimedean Property (Alternative formulation)
Theorem 59: Nested Interval Property
Example 60
Solution. Suppose by contradiction that the intersection is non-empty. Then there exists \(x \in \mathbb{N}\) such that \[ x \in I_n \,, \quad \forall \, n \in \mathbb{N}\,. \] By definition of \(I_n\) the above reads \[ 0 < x < \frac1n \,, \quad \forall \, n \in \mathbb{N}\,. \tag{A.16}\] Since \(x>0\), by the Archimedean Property in Theorem 57 Point 2, there exists \(n_0 \in \mathbb{N}\) such that \[ 0 < \frac{1}{n_0} < x \,. \] The above contradicts (A.16). Therefore (A.15) holds.
A.5.1 Revisiting Sup and Inf
Proposition 61: Characterization of Supremum
Let \(A \subseteq \mathbb{R}\) be a non-empty set. Suppose that \(s \in \mathbb{R}\) is an upper bound for \(A\). They are equivalent:
- \(s = \sup A\)
- For every \(\varepsilon> 0\) there exists \(x \in A\) such that \[ s - \varepsilon< x \,. \]
Proposition 62: Characterization of Infimum
Let \(A \subseteq \mathbb{R}\) be a non-empty set. Suppose that \(i \in \mathbb{R}\) is a lower bound for \(A\). They are equivalent:
- \(i = \inf A\)
- For every \(\varepsilon\in \mathbb{R}\), with \(\varepsilon> 0\), there exists \(x \in A\) such that \[ x < i + \varepsilon\,. \]
Proposition 63
Corollary 64
Corollary 65
Proposition 66
Proof
Part 2. We have \[ \frac1n > 0 \,, \quad \forall \, n \in \mathbb{N}\,, \] showing that \(0\) is a lower bound for \(A\). Suppose by contradiction that \(0\) is not the infimum. Therefore \(0\) is not the largest lower bound. Then there exists \(\varepsilon\in \mathbb{R}\) such that:
\(\varepsilon\) is a lower bound for \(A\), that is, \[ \varepsilon\leq \frac1n \,, \quad \forall \, n \in \mathbb{N}\,, \tag{A.17}\]
\(\varepsilon\) is larger than \(0\): \[ 0 < \varepsilon\,. \]
As \(\varepsilon>0\), by the Archimedean Property there exists \(n_0 \in \mathbb{N}\) such that \[ 0 < \frac{1}{n_0} < \varepsilon\,. \] This contradicts (A.17). Thus \(0\) is the largest lower bound of \(A\), that is, \(0 = \inf A\).
Part 3. We have that \(\min A\) does not exist. Indeed suppose by contradiction that \(\min A\) exists. Then \[ \min A = \inf A \,. \] As \(\inf A = 0\) by Part 2, we conclude \(\min A = 0\). As \(\min A \in A\), we obtain \(0 \in A\), which is a contradiction.
A.5.2 Cardinality
Definition 67: Cardinality, Finite, Countable, Uncountable
Let \(X\) be a set. The cardinality of \(X\) is the number of elements in \(X\). We denote the cardinality of \(X\) by \[ |X| := \# \, \mbox{ of elements in } \, X \,. \] Further, we say that:
\(X\) is finite if there exists a natural number \(n \in \mathbb{N}\) and a bijection \[ f \colon \{1,2,\ldots, n \} \to X \,. \] In particular \[ |X| = n \,. \]
\(X\) is countable if there exists a bijection \[ f \colon \mathbb{N}\to X \,. \] In this case we denote the cardinality of \(X\) by \[ |X| = |\mathbb{N}| \,. \]
\(X\) is uncountable if \(X\) is neither finite, nor countable.
Proposition 68
Example 69
Solution. Set \(Y=\{ 1,2,3\}\). The function \(f \colon X \to Y\) defined by \[ f(1)=a \,, \quad f(2) = b \,, \quad f(3) = c \,, \] is bijective. Therefore \(X\) is finite, with \(|X| = 3\).
Example 70
Solution. The function \(f \colon X \to \mathbb{N}\) defined by \[ f(n):=n \,, \] is bijective. Therefore \(X = \mathbb{N}\) is countable.
Example 71
Solution. Define the map \(f \colon \mathbb{N}\to X\) by \[ f(n):= 2n \,. \] We have that:
\(f\) is injective, because \[ f(m)=f(k) \, \implies \, 2m=2k \, \quad \, m=k \,. \]
\(f\) is surjective: Suppose that \(m \in X\). By definition of \(X\), there exists \(n \in \mathbb{N}\) such that \(m = 2n\). Therefore, \(f(n) = m\).
We have shown that \(f\) is bijective. Thus, \(X\) is countable.
Example 72
Solution. Define \(f \colon \mathbb{N}\to \mathbb{Z}\) by \[ f(n):= \begin{cases} \dfrac{n}{2} & \,\, \mbox{ if } \, n \, \mbox{ even } \\ -\dfrac{n+1}{2} & \,\, \mbox{ if } \, n \, \mbox{ odd } \end{cases} \] For example \[\begin{align*} f(0) & = 0 \,, \quad f(1) = -1 \,, \quad f(2) = 1 \,, \quad f(3) = -2 \,, \\ f(4) & = 2 \,, \quad f(5) = -3 \,, \quad f(6) = 3 \,, \quad f(7) = -4 \,. \end{align*}\] We have:
\(f\) is injective: Indeed, suppose that \(m \neq n\). If \(n\) and \(m\) are both even or both odd we have, respectively \[\begin{align*} f(m) = \frac{m}{2} & \neq \frac{n}{2} = f(n) \\ f(m) = -\frac{m+1}{2} & \neq - \frac{n+1}{2} = f(n) \,. \end{align*}\] If instead \(m\) is even and \(n\) is odd, we get \[ f(m) = \frac{m}{2} \neq - \frac{n+1}{2} = f(n) \,, \] since the LHS is positive and the RHS is negative. The case when \(m\) is odd and \(n\) even is similar.
\(f\) is surjective: Let \(z \in \mathbb{Z}\). If \(z \geq 0\), then \(m:=2z\) belongs to \(\mathbb{N}\), is even, and \[ f(m) = f(2z) = z \,. \] If instead \(z < 0\), then \(m := -2z -1\) belongs to \(\mathbb{N}\), is odd, and \[ f(m) = f(-2z-1) = z \,. \]
Therefore \(f\) is bijective, showing that \(\mathbb{Z}\) is countable.
Proposition 73
Theorem 74: \(\mathbb{Q}\) is countable
Theorem 75: \(\mathbb{R}\) is uncountable
Theorem 76
Proof
A.6 Complex Numbers
Definition 77: Complex Numbers
The set of complex numbers \(\mathbb{C}\) is defined as
\[ \mathbb{C}:= \mathbb{R}+ i \mathbb{R}:= \{x + i y \, \colon \,x, y \in \mathbb{R}\} \,. \] For a complex number \[ z = x + i y \in \mathbb{C} \] we say that
- \(x\) is the real part of \(z\), and denote it by \[ x = \operatorname{Re}(z) \]
- \(y\) is the imaginary part of \(z\), and denote it by \[ y = \operatorname{Im}(z) \]
We say that
- If \(\operatorname{Re}z = 0\) then \(z\) is a purely imaginary number.
- If \(\operatorname{Im}z = 0\) then \(z\) is a real number.
Definition 78: Addition and multiplication in \(\mathbb{C}\)
Let \(z_1,z_2 \in \mathbb{C}\), so that \[ z_1 = x_1 + i y_1 \,\,, \quad z_2 = x_2 + i y_2 \,\, , \] for some \(x_1,x_2,y_1,y_2 \in \mathbb{R}\):
The sum of \(z_1\) and \(z_2\) is \[ z_1 + z_2 :=\left(x_{1}+x_{2}\right) + i \left(y_{1}+y_{2}\right) \,. \]
The multiplication of \(z_1\) and \(z_2\) is \[\begin{align*} z_1 \cdot z_2 := \left(x_{1} \cdot x_{2} - y_{1} \cdot y_{2}\right) + i \left(x_{1} \cdot y_{2} + x_{2} \cdot y_{1} \right) \,, \end{align*}\]
Example 79
Solution. Using the definition we compute \[\begin{align*} z \cdot w & = (-2+3 i) \cdot (1 - i) \\ & = (-2-(-3))+(2+3) i \\ & = 1 + 5 i \, . \end{align*}\] Alternatively, we can proceed formally: We just need to recall that \(i^2\) has to be replaced with \(-1\): \[\begin{align*} z \cdot w & = (-2+3 i) \cdot (1 - i) \\ & = - 2 + 2i + 3i - 3 i^2 \\ & = (-2 + 3 ) + ( 2 + 3 ) i \\ & = 1 + 5 i \, . \end{align*}\]
Proposition 80: Additive inverse in \(\mathbb{C}\)
Proposition 81: Multiplicative inverse in \(\mathbb{C}\)
Proof
Example 82
Solution. By the formula in Propostion 81 we immediately get \[ z^{-1} = \frac{3}{3^{2}+2^{2}} + \, \frac{-2}{3^{2}+2^{2}} \, i = \frac{3}{13}-\frac{2}{13} \, i \,. \] Alternatively, we can proceed formally: \[\begin{align*} (3+2 i)^{-1} & = \frac{1}{3+2 i} \\ & = \frac{1}{3+2 i} \, \frac{3-2 i}{3-2 i} \\ & = \frac{3-2 i}{3^2+2^2} \\ & = \frac{3}{13}-\frac{2}{13} i \,, \end{align*}\] and obtain the same result.
Theorem 83
Example 84
Question. Let \(w=1+i\) and \(z=3-i\). Compute \(\frac{w}{z}\).
Solution. We compute \(w/z\) using the two options we have:
Using the formula for the inverse from Proposition 81 we compute \[\begin{align*} z^{-1} & = \frac{x}{x^{2}+y^{2}} + i \, \frac{-y}{x^{2}+y^{2}} \\ & = \frac{3}{3^2 + 1^2} - i \, \frac{-1}{3^2 + 1^2} \\ & = \frac{3}{10} + \frac{1}{10} \, i \end{align*}\] and therefore \[\begin{align*} \frac{w}{z} & = w \cdot z^{-1} \\ & = (1 + i) \, \left( \frac{3}{10} + \frac{1}{10} \, i \right) \\ & = \left(\frac{3}{10}-\frac{1}{10}\right)+\left(\frac{1}{10}+\frac{3}{10}\right) i \\ & = \frac{2}{10}+\frac{4}{10} i \\ & = \frac{1}{5}+\frac{2}{5} i \end{align*}\]
We proceed formally, using the multiplication by \(1\) trick. We have \[\begin{align*} \frac{w}{z} & = \frac{1+i}{3-i} \\ & = \frac{1+i}{3-i} \frac{3+i}{3+i} \\ & = \frac{3-1+(3+1) i}{3^2+1^2} \\ & =\frac{2}{10}+\frac{4}{10} i \\ & = \frac{1}{5}+\frac{2}{5} i \end{align*}\]
Definition 85: Complex conjugate
\[ \bar{z}=x- i y \, . \]
Theorem 86
For all \(z_1, z_2 \in \mathbb{C}\) it holds:
\(\overline{z_1 + z_2 }=\overline{z_1}+\overline{z_2}\)
\(\overline{z_1 \cdot z_2}=\overline{z_1} \cdot \overline{z_2}\)
A.6.1 The complex plane
Definition 87: Modulus
Definition 88: Distance in \(\mathbb{C}\)
Theorem 89
Example 90
Solution. The distance is \[\begin{align*} |z- w| & = |(2-4 i)-(-5+i)| \\ & = |7-5 i| \\ & =\sqrt{7^{2}+(-5)^{2}} \\ & =\sqrt{74} \end{align*}\]
Theorem 91
Let \(z, z_{1}, z_{2} \in \mathbb{C}\). Then
\(\left|z_1 \cdot z_2\right|=\left|z_{1}\right|\left|z_{2}\right|\)
\(\left|z^{n}\right|=|z|^{n}\) for all \(n \in \mathbb{N}\)
\(z \cdot \bar{z}=|z|^{2}\)
Theorem 92: Triangle inequality in \(\mathbb{C}\)
For all \(x, y, z \in \mathbb{C}\),
\(|x+y| \leq|x|+|y|\)
\(|x-z| \leq|x-y|+|y-z|\)
A.6.2 Polar coordinates
Definition 93: Argument
Example 94
Theorem 95: Polar coordinates
Definition 96: Trigonometric form
Example 97
Solution. We have \[\begin{gather*} x = \rho \cos (\theta) = \sqrt{8} \cos \left( \frac{3}{4} \pi \right) = - \sqrt{8} \cdot \frac{\sqrt{2}}{2} = -2 \\ y = \rho \sin (\theta) = \sqrt{8} \sin \left( \frac{3}{4} \pi \right) = \sqrt{8} \cdot \frac{\sqrt{2}}{2} = 2 \,. \end{gather*}\] Therefore, the cartesian form of \(z\) is \[ z = x + i y = - 2 + 2 i \,. \]
Corollary 98: Computing \(\arg(z)\)
Example 99
Solution. Using the formula for \(\arg\) in Corollary 98 we have \[\begin{align*} \arg (3+4 i) & =\arctan \left(\frac{4}{3}\right) \\ \arg (3-4 i) & =\arctan \left(-\frac{4}{3}\right) = -\arctan \left(\frac{4}{3}\right) \\ \arg (-3+4 i) & = \arctan \left(-\frac{4}{3}\right) + \pi = - \arctan \left(\frac{4}{3}\right) + \pi \\ \arg (-3-4 i) & = \arctan \left(\frac{4}{3}\right) - \pi \end{align*}\]
A.6.3 Exponential form
Theorem 100: Euler’s identity
Theorem 101
Theorem 102
Definition 103: Exponential form
Example 104
Solution. From Example 97 we know that \(z = -2+2i\) can be written in trigonometric form as \[ z = \sqrt{8} \left[ \cos \left( \frac{3}{4} \pi \right) + i \sin \left( \frac{3}{4} \pi \right) \right] \,. \] By Euler’s identity we hence obtain the exponential form \[ z=\sqrt{8} e^{i \frac{3}{4} \pi} \,. \]
Remark 105: Periodicity of exponential
Proposition 106
Example 107
Question. Compute \((-2+2 i)^{4}\).
Solution. We have two possibilities:
Use the binomial theorem: \[\begin{align*} (-2+2 i)^{4} & =(-2)^{4}+\left(\begin{array}{l} 4 \\ 1 \end{array}\right)(-2)^{3} \cdot 2 i+\left(\begin{array}{l} 4 \\ 2 \end{array}\right)(-2)^{2} \cdot(2 i)^{2} \\ & \quad +\left(\begin{array}{l} 4 \\ 3 \end{array}\right)(-2) \cdot(2 i)^{3}+(2 i)^{4} \\ & =16-4 \cdot 8 \cdot 2 i-6 \cdot 4 \cdot 4+4 \cdot 2 \cdot 8 i+16 \\ & =16-64 i-96+64 i+16=-64 \,. \end{align*}\]
A much simpler calculation is possible by using the exponential form: We know that \[ -2+2 i = \sqrt{8} e^{i \frac{3}{4} \pi} \] by Example 104. Hence \[ (-2+2 i)^{4}=\left(\sqrt{8} e^{i \frac{3}{4} \pi}\right)^{4}=8^{2} e^{i 3 \pi}=-64 \,, \] where we used that \[ e^{i 3 \pi} = e^{i \pi} = \cos(\pi) + i \sin(\pi) = - 1 \] by \(2\pi\) periodicity of \(e^{i\theta}\) and Euler’s identity.
Definition 108: Complex exponential
Theorem 109
Example 110
Solution. We know that \[ |i| = 1 \,, \quad \arg(i) = \frac{\pi}{2} \,. \] Hence we can write \(i\) in exponential form \[ i= |i| e^{i\arg(i)} = e^{i \frac{\pi}{2}} \,. \] Therefore \[ i^{i}=\left(e^{i \frac{\pi}{2}}\right)^{i}=e^{i^2\frac{\pi}{2}}=e^{-\frac{\pi}{2}} \,. \]
A.6.4 Fundamental Theorem of Algebra
Theorem 111: Fundamental theorem of algebra
Example 112
Solution. The equation \(z^2 = -1\) is equivalent to \[ p(z) = 0 \,, \quad p(z):=z^{2}+1 \,. \] Since \(p\) has degree \(n=2\), the Fundamental Theorem of Algebra tells us that there are two solutions to (A.22). We have already seen that these two solutions are \(z=i\) and \(z=-i\). Then \(p\) factorizes as \[ p(z) = z^{2} + 1 = (z-i)(z+i) \,. \]
Example 113
Solution The associated polynomial equation is \[
p(z) = 0 \,, \quad p(z) := z^4 - 1 \,.
\] Since \(p\) has degree \(n=4\), the Fundamental Theorem of Algebra tells us that there are \(4\) solutions to (A.23). Let us find such solutions. We use the well known identity \[
a^2-b^2 = (a+b)(a-b) \,, \quad \forall \, a,b \in \mathbb{R}\,,
\] to factorize \(p\). We get: \[
p(z) = (z^4-1) = (z^2+1)(z^2-1) \,.
\] We know that \[
z^2 + 1 = 0
\] has solutions \(z = \pm i\). Instead
\[
z^2 - 1 = 0
\] has solutions \(x = \pm 1\). Hence, the four solutions of (A.23) are given by \[
z=1,-1, i,-i \,,
\] and \(p\) factorizes as \[
p(z) = z^4 - 1 = (z-1)(z+1)(z-i)(z+i) \,.
\]
Definition 114
Example 115
The equation \[ (z-1)(z-2)^{2}(z+i)^{3}=0 \] has 6 solutions:
- \(z=1\) with multiplicity \(1\)
- \(z=2\) with multiplicity \(2\)
- \(z=-i\) with multiplicity \(3\)
A.6.5 Solving polynomial equations
Proposition 116: Quadratic formula
- If \(\Delta > 0\) then (A.24) has two distinct real solutions \(z_1,z_2 \in \mathbb{R}\) given by \[ z_1 = \frac{-b - \sqrt{\Delta}}{2 a} \,, \quad z_2 = \frac{-b + \sqrt{\Delta}}{2 a} \,. \]
- If \(\Delta = 0\) then (A.24) has one real solution \(z \in \mathbb{R}\) with multiplicity \(2\). Such solution is given by \[ z = z_1 = z_2 = \frac{-b}{2 a} \,. \]
- If \(\Delta < 0\) then (A.24) has two distinct complex solutions \(z_1,z_2 \in \mathbb{C}\) given by \[ z_1 = \frac{-b - i\sqrt{-\Delta}}{2 a} \,, \quad z_2 = \frac{-b + i\sqrt{-\Delta}}{2 a} \,, \] where \(\sqrt{-\Delta} \in \mathbb{R}\), since \(-\Delta>0\).
In all cases, the polynomial at (A.24) factorizes as \[ a z^{2}+b z+c = a (z-z_1)(z-z_2) \,. \]
Example 117
Question. Solve the following equations:
- \(3 z^{2}-6 z+2 = 0\)
- \(4 z^{2}-8 z+4=0\)
- \(z^{2}+2 z+3=0\)
Solution.
We have that \[ \Delta = (-6)^{2}-4 \cdot 3 \cdot 2 = 12 > 0 \] Therefore the equation has two distinct real solutions, given by \[ z=\frac{-(-6) \pm \sqrt{12}}{2 \cdot 3}=\frac{6 \pm \sqrt{12}}{6}=1 \pm \frac{\sqrt{3}}{3} \] In particular we have the factorization \[ 3 z^{2}-6 z+2 = 3 \left( z - 1 - \frac{\sqrt{3}}{3} \right) \left( z - 1 + \frac{\sqrt{3}}{3} \right) \,. \]
We have that \[ \Delta = (-8)^{2}-4 \cdot 4 \cdot 4 = 0 \,. \] Therefore there exists one solution with multiplicity \(2\). This is given by \[ z=\frac{-(-8)}{2 \cdot 4} = 1 \,. \] In particular we have the factorization \[ 4 z^{2}-8 x+4 = 4 (z-1)^2 \,. \]
We have \[ \Delta = 2^{2}-4 \cdot 1 \cdot 3 = - 8 < 0 \,. \] Therefore there are two complex solutions given by \[ z=\frac{-2 \pm i \sqrt{8}}{2 \cdot 1} = -1 \pm i \sqrt{2} \,. \] In particular we have the factorization \[ z^{2}+2 z+3 = (z + 1 - i \sqrt{2}) (z + 1 + i\sqrt{2}) \,. \]
Proposition 118: Quadratic formula with complex coefficients
Example 119
Solution. We have \[\begin{align*} \Delta & = (-(3+i))^{2}-4 \cdot \frac{1}{2} \cdot(4-i) \\ & = 8+6 i-8+2 i \\ & =8 i \,. \end{align*}\] Therefore \(\Delta \in \mathbb{C}\). We have to find solutions \(S_1\) and \(S_2\) to the equation \[ z^2 = \Delta = 8i \,. \tag{A.26}\] We look for solutions of the form \(z=a+ i b\). Then we must have that \[ z^{2}=(a+ ib)^{2}=a^{2}-b^{2}+2 a b i = 8 i \,. \] Thus \[ a^{2}-b^{2}=0 \,, \quad 2 a b = 8 \,. \] From the first equation we conclude that \(|a|=|b|\). From the second equation we have that \(ab=4\), and therefore \(a\) and \(b\) must have the same sign. Hence \(a=b\), and \[ 2 a b = 8 \quad \implies \quad a = b = \pm 2 \,. \] From this we conclude that the solutions to (A.26) are \[ S_{1} = 2+2 i \,, \quad S_{2}=-2-2 i \,. \] Hence the solutions to (A.25) are \[\begin{align*} z_1 & = \frac{3+i+S_{1}}{2 \cdot \frac{1}{2}} = 3 + i + S_{1} \\ & = 3 + i + 2 + 2i = 5 + 3i \,, \end{align*}\] and \[\begin{align*} z_2 & = \frac{3+i+S_{2}}{2 \cdot \frac{1}{2}} = 3+i+S_{2} \\ & = 3+i -2 - 2i = 1 - i \,. \end{align*}\] In particular, the given polynomial factorizes as \[\begin{align*} \frac{1}{2} z^{2}-(3+i) z+(4-i) & = \frac12 (z - z_1)(z-z_2) \\ & = \frac12 (z - 5 - 3i)(z - 1 + i) \,. \end{align*}\]
Example 120
Question. Consider the equation \[ z^{3}-7 z^{2}+6 z=0 \,. \]
- Check whether \(z=0,1,-1\) are solutions.
- Using your answer from Point 1, and polynomial division, find all the solutions.
Solution.
By direct inspection we see that \(z=0\) and \(z=1\) are solutions.
Since \(z = 0\) is a solution, we can factorize \[ z^{3}-7 z^{2}+6 z=z\left(z^{2}-7 z+6\right) \,. \] We could now use the quadratic formula on the term \(z^{2}-7 z+6\) to find the remaining two roots. However, we have already observed that \(z=1\) is a solution. Therefore \(z-1\) divides \(z^{2}-7 z+6\). Using polynomial long division, we find that \[ \frac{z^{2}-7 z+6}{z-1}=z-6 \,. \] Therefore the last solution is \(z=6\), and \[ z^{3}-7 z^{2}+6 z=z(z-1)(z-6) \,. \]
Example 121
Solution. It is easy to see \(z=1\) is a solution. This means that \(z-1\) divides \(z^{3}-7 z+6\). By using polynomial long division, we compute that \[ \frac{z^{3}-7 z+6}{z-1}=z^{2}+z-6 \,. \] We are now left to solve \[ z^{2}+z-6 = 0\,. \] Using the quadratic formula, we see that the above is solved by \(z=2\) and \(z=-3\). Therefore the given polynomial factorizes as \[ z^{3}-7 z+6 = (z-1)(z-2)(z+3) \,. \]
A.6.6 Roots of unity
Theorem 122
Definition 123
Example 124
Solution. The \(4\) solutions are given by \[ z_k = \exp \left(i \frac{2 \pi k}{4} \right) = \exp \left(i \frac{\pi k}{2} \right) \,, \] for \(k=0,1,2,3\). We compute: \[\begin{align*} z_0 & = e^{i 0} = 1 \,, & \quad & z_1 = e^{i \frac{\pi}{2}}=i \,, \\ z_2 & = e^{i \pi}=-1 \,, & \quad & z_3 = e^{i \frac{3 \pi}{2}}=-i \, . \end{align*}\] Note that for \(k=4\) we would again get the solution \(z=e^{i 2 \pi}=1\).
Example 125
Solution. The \(3\) solutions are given by \[ z_k = \exp \left( i \frac{2 \pi k}{3} \right) \,, \] for \(k=0,1,2\). We compute: \[ z_0=e^{i 0}=1, \quad z_1=e^{i \frac{2 \pi}{3}}, \quad z_2=e^{i \frac{4 \pi}{3}} . \] We can write \(z_1\) and \(z_2\) in cartesian form: \[ z_1 = e^{i \frac{2 \pi}{3}}=\cos \left(\frac{2 \pi}{3}\right)+i \sin \left(\frac{2 \pi}{3}\right)=-\frac{1}{2}+\frac{\sqrt{3}}{2} i \] and \[ z_2 = e^{i \frac{4 \pi}{3}}=\cos \left(\frac{4 \pi}{3}\right)+i \sin \left(\frac{4 \pi}{3}\right)=-\frac{1}{2}-\frac{\sqrt{3}}{2} i \,. \]
A.6.7 Roots in \(\mathbb{C}\)
Theorem 126
Example 127
Solution. Let \(c = -32\). We have \[ |c| = |-32|=32=2^{5}\,, \quad \theta = \arg (-32)=\pi \,. \] The \(5\) solutions are given by \[ z_k = \left(2^{5}\right)^{\frac{1}{5}} \exp \left(i \pi \, \frac{1+2 k}{5} \right) \,, \quad k \in \mathbb{Z}\,, \] for \(k=0,1,2,3,4\). We get \[\begin{align*} z_0 & = 2 e^{i \frac{\pi}{5}} \, & \quad & z_1 = 2 e^{i \frac{3 \pi}{5}} \\ z_2 & = 2 e^{i \pi}=-2 \, & \quad & z_3=2 e^{i \frac{7 \pi}{5}} \\ z_4 &= 2 e^{i \frac{9 \pi}{5}} & \quad & \end{align*}\]
Example 128
Solution. Set \[ c:=9\left(\cos \left(\frac{\pi}{3}\right)+i \sin \left(\frac{\pi}{3}\right)\right) \,. \] The complex number \(c\) is already in the trigonometric form, so that we can immediately obtain \[ |c| = 9 \,, \quad \theta = \arg(c) = \frac{\pi}{3} \,. \] The \(4\) solutions are given by \[\begin{align*} z_k & = \sqrt[4]{9} \, \exp \left( i \, \frac{ \pi/3 + 2 \pi k}{4} \right) \\ & = \sqrt{3} \exp \left( i \pi \, \frac{1+6 k}{12} \right) \end{align*}\] for \(k=0,1,2,3\). We compute \[\begin{align*} z_0 & = \sqrt{3} e^{i \pi \frac{1}{12}} & \quad & z_1 = \sqrt{3} e^{i \pi \frac{7}{12}} \\ z_2 & = \sqrt{3} e^{i \pi \frac{13}{12}} & \quad & z_3 = \sqrt{3} e^{i \pi \frac{19}{12}} \end{align*}\]
A.7 Sequences in \(\mathbb{R}\)
Definition 129: Convergent sequence
Theorem 130
Proof
Example 131
Question. Using the definition of convergence, prove that \[ \lim_{n \to \infty} \frac{n}{2n+3} = \frac{1}{2} \,. \]
Solution.
Rough Work: Let \(\varepsilon>0\). We want to find \(N \in \mathbb{N}\) such that \[ \left|\frac{n}{2n+3}-\frac{1}{2}\right| < \varepsilon\,, \quad \forall \, n \geq N \,. \] To this end, we compute: \[\begin{align*} \left|\frac{n}{2 n+3}-\frac{1}{2}\right| & =\left|\frac{2n -(2n + 3)}{2(2n +3)}\right| \\ & =\left|\frac{- 3}{4n + 6}\right| \\ & = \frac{3}{4n + 6} \,. \end{align*}\] Therefore \[\begin{align*} \left|\frac{n}{2n+3}-\frac{1}{2}\right| < \varepsilon \quad \iff & \quad \frac{3}{4n + 6} < \varepsilon\\ \quad \iff & \quad \frac{4n + 6}{3} > \frac{1}{\varepsilon} \\ \quad \iff & \quad 4n + 6 > \frac{3}{\varepsilon} \\ \quad \iff & \quad 4n > \frac{3}{\varepsilon} - 6 \\ \quad \iff & \quad n > \frac{3}{4\varepsilon} - \frac{6}{4} \,. \end{align*}\] Looking at the above equivalences, it is clear that \(N \in \mathbb{N}\) has to be chosen so that \[ N > \frac{3}{4\varepsilon} - \frac{6}{4} \,. \]
Formal Proof: We have to show that \[ \forall \varepsilon>0 \,, \, \exists \, N \in \mathbb{N}\, \text{ s.t. } \, \forall \, n \geq N \,, \, \left|\frac{n}{2n+3}-\frac{1}{2}\right| < \varepsilon\,. \] Let \(\varepsilon>0\). Choose \(N \in \mathbb{N}\) such that \[ N > \frac{3}{4\varepsilon} - \frac{6}{4} \,. \tag{A.30}\] By the rough work shown above, inequality (A.30) is equivalent to \[ \frac{3}{4N + 6} < \varepsilon\,. \] Let \(n \geq N\). Then \[\begin{align*} \left|\frac{n}{2n+3}-\frac{1}{2}\right| & = \frac{3}{4 n+ 6 } \\ & \leq \frac{3}{4 N+ 6 } \\ & < \varepsilon\,, \end{align*}\] where in the third line we used that \(n \geq N\).
Definition 132: Divergent sequence
Theorem 133
Proof
Theorem 134: Uniqueness of limit
Definition 135: Bounded sequence
Theorem 136
Example 137
Corollary 138
Remark 139
Theorem 140
Theorem 141
Theorem 142: Algebra of limits
Let \(\left(a_{n}\right)_{n \in \mathbb{N}}\) and \(\left(b_{n}\right)_{n \in \mathbb{N}}\) be sequences in \(\mathbb{R}\). Suppose that \[ \lim_{n \rightarrow \infty} a_{n}=a \,, \quad \lim_{n \rightarrow \infty} b_{n}=b \,, \] for some \(a,b \in \mathbb{R}\). Then,
Limit of sum is the sum of limits: \[ \lim_{n \rightarrow \infty}\left(a_{n} \pm b_{n}\right)=a \pm b \]
Limit of product is the product of limits: \[ \lim _{n \rightarrow \infty}\left(a_{n} b_{n}\right) = a b \]
If \(b_{n} \neq 0\) for all \(n \in \mathbb{N}\) and \(b \neq 0\), then \[ \lim_{n \rightarrow \infty} \left(\frac{a_{n}}{b_{n}}\right)=\frac{a}{b} \]
Example 143
Solution. We can rewrite \[ \frac{3 n}{7 n+4}=\frac{3}{7+\dfrac{4}{n}} \] From Theorem 130, we know that \[ \frac{1}{n} \rightarrow 0 \,. \] Hence, it follows from Theorem 143 Point 2 that \[ \frac{4}{n} = 4 \cdot \frac1n \rightarrow 4 \cdot 0 = 0 \,. \] By Theorem 143 Point 1 we have \[ 7 + \frac{4}{n} \rightarrow 7 + 0 = 7 \,. \] Finally we can use Theorem 143 Point 3 to infer \[ \frac{3 n}{7 n+4}=\frac{3}{7+\dfrac{4}{n}} \rightarrow \frac{3}{7} \,. \]
Example 144
Solution. Factor \(n^2\) to obtain \[ \frac{n^{2}-1}{2 n^{2}-3} = \frac{1-\dfrac{1}{n^{2}}}{2-\dfrac{3}{n^{2}}} \,. \] By Theorem 130 we have \[ \frac{1}{n^2} \to 0 \,. \] We can then use the Algebra of Limits Theorem 143 Point 2 to infer \[ \frac{3}{n^2} \to 3 \cdot 0 = 0 \] and Theorem 143 Point 1 to get \[ 1 - \frac{1}{n^2} \to 1 - 0 = 1 \,, \quad 2 - \frac{3}{n^2} \to 2 - 0 = 2 \,. \] Finally we use Theorem 143 Point 3 and conclude \[ \frac{1-\dfrac{1}{n^{2}}}{2-\dfrac{3}{n^{2}}} \to \frac{1}{2} \,. \] Therefore \[ \lim_{n \to \infty } \, \frac{n^{2}-1}{2 n^{2}-3} = \lim_{n \to \infty} \, \frac{1-\dfrac{1}{n^{2}}}{2-\dfrac{3}{n^{2}}} = \frac{1}{2} \,. \]
Example 145
Solution. To show that the sequence \(\left(a_n\right)\) does not converge, we divide by the largest power in the denominator, which in this case is \(n^2\) \[\begin{align*} a_n & = \frac{4 n^{3}+8 n+1}{7 n^{2}+2 n+1} \\ & =\frac{4 n+ \dfrac{8}{n} + \dfrac{1}{n^{2}} }{7+ \dfrac{2}{n} + \dfrac{1}{n^{2}} } \\ & = \frac{b_n}{c_n} \end{align*}\] where we set \[ b_n := 4 n+ \dfrac{8}{n} + \dfrac{1}{n^{2}}\,, \quad c_n := 7 + \dfrac{2}{n} + \dfrac{1}{n^{2}} \,. \] Using the Algebra of Limits Theorem 143 we see that \[ c_n = 7+ \dfrac{2}{n} + \dfrac{1}{n^{2}} \to 7 \,. \] Suppose by contradiction that \[ a_n \to a \] for some \(a \in \mathbb{R}\). Then, by the Algebra of Limits, we would infer \[ b_n = c_n \cdot a_n \to 7 a \,, \] concluding that \(b_n\) is convergent to \(7a\). We have that \[ b_n = 4n + d_n \,, \quad d_n := \dfrac{8}{n} + \dfrac{1}{n^{2}} \,. \] Again by the Algebra of Limits Theorem 143 we get that \[ d_n = \dfrac{8}{n} + \dfrac{1}{n^{2}} \to 0 \,, \] and hence \[ 4n = b_n - d_n \to 7a - 0 = 7a \,. \] This is a contradiction, since the sequence \((4n)\) is unbounded, and hence cannot be convergent. Hence \((a_n)\) is not convergent.
Example 146
Solution. The first fraction in \((a_n)\) does not converge, as it is unbounded. Therefore we cannot use Point 2 in Theorem 143 directly. However, we note that \[\begin{align*} a_{n} & =\frac{2 n^{3}+7 n+1}{5 n+9} \cdot \frac{8 n+9}{6 n^{3}+8 n^{2}+3} \\ & = \frac{8 n+9}{5 n+9} \cdot \frac{2 n^{3}+7 n+1}{6 n^{3}+8 n^{2}+3} \,. \end{align*}\] Factoring out \(n\) and \(n^3\), respectively, and using the Algebra of Limits, we see that \[ \frac{8 n+9}{5 n+9}=\frac{8+9 / n}{5+9 / n} \to \frac{8+0}{5+0}=\frac{8}{5} \] and \[ \frac{2+7 / n^{2}+1 / n^{3}}{6+8 / n+3 / n^{3}} \to \frac{2+0+0}{6+0+0}=\frac{1}{3} \] Therefore Theorem 143 Point 2 ensures that \[ a_{n} \to \frac{8}{5} \cdot \frac{1}{3}=\frac{8}{15} \,. \]
Example 147
Solution. The largest power of \(n\) in the denominator is \(n^{3/2}\). Hence we factor out \(n^{3/2}\) \[\begin{align*} a_n & = \frac{n^{7 / 3}+2 \sqrt{n}+7}{4 n^{3 / 2}+5 n} \\ & = \frac{n^{7 / 3-3 / 2}+2 n^{1/2 - 3/2} + 7 n^{-3 / 2}}{4 + 5 n^{-3/2} } \\ & = \frac{n^{5/6}+2 n^{- 1} + 7 n^{-3 / 2}}{4 + 5 n^{-3/2} } \\ & = \frac{b_n}{c_n} \end{align*}\] where we set \[ b_n := n^{5/6}+2 n^{- 1} + 7 n^{-3 / 2} \,, \quad c_n := 4 + 5 n^{-3/2} \,. \] We see that \(b_n\) is unbounded while \(c_n \to 4\). By the Algebra of Limits (and usual contradiction argument) we conclude that \((a_n)\) is divergent.
Theorem 148
Example 149
Solution. We first rewrite \[\begin{align*} a_n & = \sqrt{9 n^{2}+3 n+1}-3 n \\ & = \frac{\left(\sqrt{9 n^{2}+3 n+1}-3 n\right)\left(\sqrt{9 n^{2}+3 n+1}+3 n\right)}{\sqrt{9 n^{2}+3 n+1}+3 n} \\ & =\frac{9 n^{2}+3 n+1-(3 n)^{2}}{\sqrt{9 n^{2}+3 n+1}+3 n} \\ & =\frac{3 n+1}{\sqrt{9 n^{2}+3 n+1}+3 n} \, . \end{align*}\] The biggest power of \(n\) in the denominator is \(n\). Therefore we factor out \(n\): \[\begin{align*} a_n & = \sqrt{9 n^{2}+3 n+1}-3 n \\ & =\frac{3 n+1}{\sqrt{9 n^{2}+3 n+1}+3 n} \\ & =\frac{3+ \dfrac{1}{n} }{\sqrt{9+ \dfrac{3}{n} + \dfrac{1}{n^{2}}} + 3 } \,. \end{align*}\] By the Algebra of Limits we have \[ 9+ \frac{3}{n} + \frac{1}{n^{2}} \to 9 + 0 + 0 = 9 \,. \] Therefore we can use Theorem 148 to infer \[ \sqrt{ 9 + \frac{3}{n} + \frac{1}{n^{2}} } \to \sqrt{9} \,. \] By the Algebra of Limits we conclude: \[ a_n = \frac{3+ \dfrac{1}{n} }{\sqrt{9+ \dfrac{3}{n} + \dfrac{1}{n^{2}} }+ 3 } \to \frac{ 3 + 0 }{ \sqrt{9} + 3 } = \frac12 \,. \]
Example 150
Solution. We rewrite \(a_n\) as \[\begin{align*} a_n & = \sqrt{9 n^{2}+3 n+1}-2 n \\ & =\frac{ (\sqrt{9 n^{2}+3 n+1} - 2 n) (\sqrt{9 n^{2}+3 n+1}+2 n) }{\sqrt{9 n^{2}+3 n+1}+2 n} \\ & =\frac{9 n^{2}+3 n+1-(2 n)^{2}}{\sqrt{9 n^{2}+3 n+1}+2 n} \\ & =\frac{5 n^{2}+3 n+1}{\sqrt{9 n^{2}+3 n+1}+2 n} \\ & =\frac{5 n + 3 + \dfrac{1}{n} }{\sqrt{9+ \dfrac{3}{n} + \dfrac{1}{ n^2 } } + 2 } \\ & = \frac{b_n}{c_n} \,, \end{align*}\] where we factored \(n\), being it the largest power of \(n\) in the denominator, and defined \[ b_n : = 5 n + 3 + \dfrac{1}{n}\,, \quad c_n := \sqrt{9+ \dfrac{3}{n} + \dfrac{1}{ n^2 } } + 2 \,. \] Note that \[ 9+ \dfrac{3}{n} + \dfrac{1}{ n^2 } \to 9 \] by the Algebra of Limits. Therefore \[ \sqrt{9+ \dfrac{3}{n} + \dfrac{1}{ n^2 }} \to \sqrt{9} = 3 \] by Theorem 148. Hence \[ c_n = \sqrt{9+ \dfrac{3}{n} + \dfrac{1}{ n^2 }} + 2 \to 3 + 2 = 5 \,. \] The numerator \[ b_n = 5 n + 3 + \dfrac{1}{n} \] is instead unbounded. Therefore \((a_n)\) is not convergent, by the Algebra of Limits and the usual contradiction argument.
A.8 Limit Tests
In this section we discuss a number of Tests to determine whether a sequence converges or not. These are known as Limit Tests.
A.8.1 Squeeze Theorem
When a sequence \((a_n)\) oscillates, it is difficult to compute the limit. Examples of terms which produce oscillations are \[ (-1)^n \,, \quad \sin(n) \,, \quad \cos(n) \,. \] In such instance it might be useful to compare \((a_n)\) with other sequences whose limit is known. If we can prove that \((a_n)\) is squeezed between two other sequences with the same limiting value, then we can show that also \((a_n)\) converges to this value.
Theorem 151: Squeeze theorem
Proof
Example 152
Solution. For all \(n \in \mathbb{N}\) we can estimate \[ -1 \leq(-1)^{n} \leq 1 \,. \] Therefore \[ \frac{-1}{n} \leq \frac{(-1)^{n}}{n} \leq \frac{1}{n} \,, \quad \forall \, n \in \mathbb{N}\,. \] Moreover \[ \lim_{n \to \infty} \frac{-1}{n}= -1 \cdot 0=0 \,, \quad \lim_{n \to \infty} \frac{1}{n}=0 \,. \] By the Squeeze Theorem 151 we conclude \[ \lim_{n \to \infty} \frac{(-1)^{n}}{n}=0 \,. \]
Example 153
Solution. We know that \[ -1 \leq \cos(x) \leq 1 \,, \quad - 1 \leq \sin(x) \leq 1 \,, \quad \forall \, x \in \mathbb{R}\,. \] Therefore, for all \(n \in \mathbb{N}\) \[ - 1 \leq \cos(3n) \leq 1 \,, \quad -1 \leq \sin(17n) \leq 1 \,. \] We can use the above to estimate the numerator in the given sequence: \[ -1 + 9 n^{2} \leq \cos (3 n)+9 n^{2} \leq 1+ 9 n^{2} \,. \tag{A.31}\] Concerning the denominator, we have \[ 11 n^{2}-15 \leq 11 n^{2}+15 \sin (17 n) \leq 11 n^{2} + 15 \] and therefore \[ \frac{1}{11 n^{2} + 15} \leq \frac{1}{11 n^{2}+15 \sin (17 n)} \leq \frac{1}{11 n^{2}-15} \,. \tag{A.32}\] Putting together (A.31)-(A.32) we obtain \[ \frac{-1 + 9 n^{2}}{11 n^{2} + 15} \leq \frac{\cos (3 n)+9 n^{2}}{11 n^{2}+15 \sin (17 n)} \leq \frac{1+ 9 n^{2}}{11 n^{2}-15} \,. \] By the Algebra of Limits we infer \[ \frac{-1+9 n^{2}}{11 n^{2}+15}=\frac{-\dfrac{1}{n^{2}} + 9}{11 + \dfrac{15}{n^{2}}} \to \frac{0+9}{11+0}=\frac{9}{11} \] and \[ \frac{1+9 n^{2}}{11 n^{2} - 15}=\frac{ \dfrac{1}{n^{2}} + 9}{ 11 - \dfrac{15}{n^{2}}} \to \frac{0+9}{11+0}=\frac{9}{11} \,. \] Applying the Squeeze Theorem 151 we conclude \[ \lim_{n \to \infty} \frac{\cos (3 n)+ 9 n^{2}}{11 n^{2}+15 \sin (17 n)}=\frac{9}{11} \,. \]
Warning
Example 154
Proof. Suppose by contradiction that \(a_n \to a\). We have \[ a_n = (-1)^n + \frac{(-1)^n}{n} = b_n + c_n \] where \[ b_n := (-1)^n \,, \quad c_n := \frac{(-1)^n}{n} \,. \] We have seen in Example 153 that \(c_n \to 0\). Therefore, by the Algebra of Limits, we have \[ b_n = a_n - c_n \longrightarrow a - 0 = a \,. \] However, Theorem 133 says that the sequence \(b_n = (-1)^n\) diverges. Contradiction. Hence \((a_n)\) diverges.
A.8.2 Geometric sequences
Definition 155
The value of \(|x|\) determines whether or not a geometric sequence converges, as shown in the following theorem.
Theorem 156: Geometric Sequence Test
Let \(x \in \mathbb{R}\) and let \(\left(a_{n}\right)\) be the sequence defined by \[ a_{n}:=x^{n} \,. \] We have:
If \(|x|<1\), then \[ \lim_{n \to \infty} a_{n} = 0 \,. \]
If \(|x|>1\), then sequence \(\left(a_{n}\right)\) is unbounded, and hence divergent.
Warning
The Geometric Sequence Test in Theorem 156 does not address the case \[ |x|=1 \,. \] This is because, in this case, the sequence \[ a_n = x^n \] might converge or diverge, depending on the value of \(x\). Indeed, \[ |x| = 1 \quad \implies \quad x = \pm 1 \,. \] We therefore have two cases:
\(x = 1\): Then \[ a_n = 1^n = 1 \] so that \(a_n \to 1\) and \((a_n)\) is convergent.
\(x=-1\): Then \[ a_n = x^n = (-1)^n \] which is divergent by Theorem 133.
To prove Theorem 156 we need the following inequality, known as Bernoulli’s inequality.
Lemma 157: Bernoulli’s inequality
Proof
Base case: (A.33) holds with equality when \(n=1\).
Induction hypothesis: Let \(k \in \mathbb{N}\) and suppose that (A.33) holds for \(n=k\), i.e., \[ (1+x)^{k} \geq 1+k x \,. \] Then \[\begin{align*} (1+x)^{k+1} & = (1+x)^{k}(1+x) \\ & \geq(1+k x)(1+x) \\ & =1+k x+x+k x^{2} \\ & \geq 1+(k+1) x \,, \end{align*}\] where we used that \(kx^2 \geq 0\). Then (A.33) holds for \(n=k+1\).
By induction we conclude (A.33).
We are ready to prove Theorem 156.
Proof: Proof of Theorem 156
If \(x=0\), then \[ a_n = x^n = 0 \] so that \(a_n \to 0\). Hence assume \(x \neq 0\). We need to prove that \[ \forall \, \varepsilon> 0 \,, \, \exists \, N \in \mathbb{N}\, \text{ s.t. } \, \forall \, n \geq N \,, \,\, |x^n - 0| < \varepsilon\,. \] Let \(\varepsilon>0\). We have \[ |x| < 1 \quad \implies \quad \frac{1}{|x|} > 1 \,. \] Therefore \[ |x|= \frac{1}{1+u} \,, \quad u:=\frac{1}{|x|} - 1 > 0 \,. \] Let \(N \in \mathbb{N}\) be such that \[ N > \frac{1}{\varepsilon u} \,, \] so that \[ \frac{1}{N u} < \varepsilon\,. \] Let \(n \geq N\). Then \[\begin{align*} \left|x^{n}-0\right| & =|x|^{n} \\ & = \left(\frac{1}{1+u}\right)^{n} \\ & =\frac{1}{(1+u)^{n}} \\ & \leq \frac{1}{1+n u} \\ & \leq \frac{1}{n u} \\ & \leq \frac{1}{N u} \\ & < \varepsilon\,, \end{align*}\] where we used Bernoulli’s inequality (A.33) in the first inequality.
Part 2. The case \(|x|>1\).
To prove that \((a_n)\) does not converge, we prove that it is unbounded. This means showing that \[ \forall \, M > 0 \,, \, \exists n \in \mathbb{N}\, \text{ s.t. } \, \left| a_{n}\right| >M \,. \] Let \(M > 0\). We have two cases:
\(0 < M \leq 1\): Choose \(n=1\). Then \[ \left|a_{1}\right|=|x|>1 \geq M \,. \]
\(M>1\): Choose \(n \in \mathbb{N}\) such that \[ n> \frac{\log M}{\log |x|} \,. \] Note that \(\log |x|>0\) since \(|x|>1\). Therefore \[\begin{align*} n>\frac{\log M}{\log |x|} & \iff n \log |x|>\log M \\ & \iff \log |x|^n>\log M \\ & \iff |x|^{n}>M \,. \end{align*}\] Then \[ \left|a_{n}\right|=\left|x^{n}\right|=|x|^{n} > M \,. \]
Hence \((a_n)\) is unbounded. By Corollary 138 we conclude that \((a_n)\) is divergent.
Example 158
We can apply Theorem 156 to prove convergence or divergence for the following sequences.
We have \[ \left(\frac{1}{2}\right)^{n} \longrightarrow 0 \] since \[ \left|\frac{1}{2}\right|=\frac{1}{2}<1 \,. \]
We have \[ \left(\frac{-1}{2}\right)^{n} \longrightarrow 0 \] since \[ \left|\frac{-1}{2}\right|=\frac{1}{2}<1 \,. \]
The sequence \[ a_n = \left(\frac{-3}{2}\right)^{n} \] does not converge, since \[ \left|\frac{-3}{2}\right|=\frac{3}{2}>1 \,. \]
As \(n \rightarrow \infty\), \[ \frac{3^{n}}{(-5)^{n}}=\left(-\frac{3}{5}\right)^{n} \longrightarrow 0 \] since \[ \left|-\frac{3}{5}\right|=\frac{3}{5}<1 \,. \]
The sequence \[ a_{n}=\frac{(-7)^{n}}{2^{2 n}} \] does not converge, since \[ \frac{(-7)^{n}}{2^{2 n}}=\frac{(-7)^{n}}{\left(2^{2}\right)^{n}}=\left(-\frac{7}{4}\right)^{n} \] and \[ \left|-\frac{7}{4}\right|=\frac{7}{4}>1 \,. \]
A.8.3 Ratio Test
Theorem 159: Ratio Test
Let \(\left(a_{n}\right)\) be a sequence in \(\mathbb{R}\) such that \[ a_{n} \neq 0 \,, \quad \forall \, n \in \mathbb{N}\,. \]
Suppose that the following limit exists: \[ L:=\lim_{n \to \infty}\left|\frac{a_{n+1}}{a_{n}}\right| \,. \] Then,
If \(L<1\) we have \[ \lim_{n \to\infty} a_{n}=0 \,. \]
If \(L>1\), the sequence \(\left(a_{n}\right)\) is unbounded, and hence does not converge.
Suppose that there exists \(N \in \mathbb{N}\) and \(L>1\) such that \[ \left|\frac{a_{n+1}}{a_{n}}\right| \geq L \,, \quad \forall \, n \geq N \,. \] Then the sequence \(\left(a_{n}\right)\) is unbounded, and hence does not converge.
Proof
\[ \left|\frac{a_{n+1}}{a_{n}}\right|=\frac{\left|a_{n+1}\right|}{\left|a_{n}\right|}=\frac{b_{n+1}}{b_{n}} \]
Part 1. Suppose that there exists the limit \[ L:=\lim_{n \to \infty}\left|\frac{a_{n+1}}{a_{n}}\right| \,. \] Therefore \[ \lim_{n \to \infty} \frac{b_{n+1}}{b_{n}} = L \,. \tag{A.34}\]
\(L<1\): Choose \(r>0\) such that \[ L<r<1 \,. \] Set \[ \varepsilon:= r-L \] By the convergence at (A.34) there exists \(N \in \mathbb{N}\) such that \[ \left|\frac{b_{n+1}}{b_{n}}-L\right| < \varepsilon= r - L \,, \quad \forall \, n \geq N \,. \] In particular \[ \frac{b_{n+1}}{b_{n}}-L < r-L \,, \quad \forall \, n \geq N \,, \] which implies \[ b_{n+1} < r \,{b_{n}} \,, \quad \forall \, n \geq N \,. \tag{A.35}\] Let \(n \geq N\), we can use (A.35) recursively and obtain \[ 0 \leq b_{n} < \, r b_{n-1} < \ldots < r^{n-N} \, b_{N} = r^{n} \, \frac{b_{N}}{r^{N}} \,. \] In particular, we have proven that \[ 0 \leq b_{n} < r^{n} \, \frac{b_{N}}{r^{N}} \,, \quad \forall \, n \in \mathbb{N}\,. \tag{A.36}\] Since \(|r|<1\), by the Geometric Sequence Test Theorem 156 we infer \[ r^{n} \rightarrow 0 \,. \] The Algebra of Limits the yields \[ r^{n} \, \frac{b_{N}}{r^{N}} \rightarrow 0 \,. \] By the Squeeze Theorem 151 applied to (A.36), it follows that \[ b_{n} = |a_n| \to 0 \,. \] Since \[ -\left|a_{n}\right| \leq a_{n} \leq\left|a_{n}\right| \,, \] and \[ -|a_n| \to 0 \,, \quad |a_n| \to 0 \,, \] we can again apply the Squeeze Theorem 151 to infer \[ a_{n} \rightarrow 0 \,. \]
\(L>1\): Choose \(r>0\) such that \[ 1<r<L \,. \] Define \[ \varepsilon:= L - r > 0 \,. \] By the convergence (A.34), there exists \(N \in \mathbb{N}\) such that \[ \left|\frac{b_{n+1}}{b_{n}}-L\right| < \varepsilon= L - r \,, \quad \forall \, n \geq N \,. \] In particular, \[ -(L-r) < \frac{b_{n+1}}{b_{n}}-L \,, \quad \forall \, n \geq N \,, \] which implies \[ b_{n+1} > r \, b_{n} \,, \quad \forall \, n \geq N \,. \tag{A.37}\] Let \(n \geq N\). Applying (A.37) recursively we get \[ b_{n} > r^{n-N} \, b_{N} = r^{n} \, \frac{b_{N}}{r^{N}} \,, \quad \forall \, n \geq N \,. \tag{A.38}\] Since \(|r|>1\), by the Geometric Sequence Test we have that the sequence \[ (r^n) \] is unbounded. Therefore also the right hand side of (A.38) is unbounded, proving that \((b_n)\) is unbounded. Since \[ b_n = |a_n|\,, \] we conclude that \((a_n)\) is unbounded. By Corollary 138 we conclude that \((a_n)\) does not converge.
Part 2. Suppose that there exists \(N \in \mathbb{N}\) and \(L>1\) such that \[ \left|\frac{a_{n+1}}{a_{n}}\right| \geq L \,, \quad \forall \, n \geq N \,. \] Since \(b_n = |a_n|\), we infer \[ b_{n+1} \geq L \, b_n \,, \quad \forall \, n \geq N \,. \] Arguing as above, we obtain \[ b_n \geq L^n \, \frac{b_N}{L^N} \,, \quad \forall \, n \geq N \,. \] Since \(L>1\), we have that the sequence \[ L^n \, \frac{b_N}{L^N} \] is unbounded, by the Geometric Sequence Test. Hence also \((b_n)\) is unbounded, from which we conclude that \((a_n)\) is unbounded. By Corollary 138 we conclude that \((a_n)\) does not converge.
Let us apply the Ratio Test to some concrete examples.
Example 160
Solution. We have \[\begin{align*} \left|\frac{a_{n+1}}{a_{n}}\right| & = \dfrac{\left( \dfrac{3^{n+1}}{(n+1) !} \right) }{ \left( \dfrac{3^{n}}{n !} \right) } \\ & = \frac{3^{n+1}}{3^{n}} \, \frac{n !}{(n+1) !} \\ & = \frac{3 \cdot 3^n}{3^n} \, \frac{n!}{(n+1) n!} \\ & =\frac{3}{n+1} \longrightarrow L = 0 \,. \end{align*}\] Hence, \(L=0<1\) so \(a_{n} \to 0\) by the Ratio Test in Theorem 159.
Example 161
Solution. We have \[\begin{align*} \left|\frac{a_{n+1}}{a_{n}}\right| & =\frac{(n+1) ! \cdot 3^{n+1}}{\sqrt{(2(n+1)) !}} \frac{\sqrt{(2 n) !}}{n ! \cdot 3^{n}} \\ & =\frac{(n+1) !}{n !} \cdot \frac{3^{n+1}}{3^{n}} \cdot \frac{\sqrt{(2 n) !}}{\sqrt{(2(n+1)) !}} \end{align*}\] For the first two fractions we have \[ \frac{(n+1) !}{n !} \cdot \frac{3^{n+1}}{3^{n}} = 3(n+1) \,, \] while for the third fraction \[\begin{align*} \frac{\sqrt{(2 n) !}}{\sqrt{(2(n+1)) !}} & =\sqrt{\frac{(2 n) !}{(2 n+2) !}} \\ & = \sqrt{\frac{ (2n)! }{ (2n+2) \cdot (2n+1) \cdot (2n)! }} \\ & = \frac{1}{\sqrt{(2 n+1)(2 n+2)}} \,. \end{align*}\] Therefore, using the Algebra of Limits, \[\begin{align*} \left|\frac{a_{n+1}}{a_{n}}\right| & = \frac{3(n+1)}{\sqrt{(2 n+1)(2 n+2)}}\\ & = \frac{3n \left(1+ \dfrac{1}{n} \right)}{\sqrt{n^2 \left( 2 + \dfrac{1}{n}\right) \left(2 + \dfrac{2}{n} \right)}} \\ & = \frac{3 \left(1+ \dfrac{1}{n} \right)}{\sqrt{ \left( 2 + \dfrac{1}{n}\right) \left(2 + \dfrac{2}{n} \right)}} \longrightarrow \frac{3}{\sqrt{4}} = \frac{3}{2} > 1 \,. \end{align*}\] By the Ratio Test we conclude that \((a_n)\) is divergent.
Example 162
Solution. We have \[ \left|\frac{a_{n+1}}{a_{n}}\right| = \frac{100^{n}}{100^{n+1}} \frac{(n+1) !}{n !} =\frac{n+1}{100} \,. \] Choose \(N=101\). Then for all \(n \geq N\), \[\begin{align*} \left|\frac{a_{n+1}}{a_{n}}\right| & = \frac{n+1}{100} \\ & \geq \frac{N+1}{100} \\ & = \frac{101}{100} > 1 \,. \end{align*}\] Hence \(a_{n}\) is divergent by the Ratio Test.
Warning
The Ratio Test in Theorem 159 does not address the case \[ L=1 \,. \] This is because, in this case, the sequence \((a_n)\) might converge or diverge.
For example:
Define the sequence \[ a_n = \frac1n \,. \] We have \[ \left|\frac{a_{n+1}}{a_{n}}\right| = \frac{n}{n+1} \rightarrow L = 1 \,. \] Hence we cannot apply the Ratio Test. However we know that \[ \lim_{n \to \infty} \, \frac1n = 0 \,. \]
Consider the sequence \[ a_n = n \,. \] We have \[ \frac{|a_{n+1}|}{|a_n|} = \frac{n+1}{n} \rightarrow L = 1 \,. \] Hence we cannot apply the Ratio Test. However we know that \((a_n)\) is unbounded, and thus divergent.
If the sequence \((a_n)\) is geometric, the Ratio Test of Theorem 159 will give the same answer as the Geometric Sequence Test of Theorem 156. This is the content of the following remark.
Remark 163
- If \(|x|<1\), the sequence \((a_n)\) converges by the Ratio Test
- If \(|x|>1\), the sequence \((a_n)\) diverges by the Ratio Test.
- If \(|x|=1\), the sequence \((a_n)\) might be convergent or divergent.
These results are in agreement with the Geometric Sequence Test.
A.9 Monotone sequences
We have seen in Theorem 136 that convergent sequences are bounded. We noted that the converse statement is not true. For example the sequence \[ a_n = (-1)^n \] is bounded but not convergent, as shown in Theorem 133. On the other hand, if a bounded sequence is monotone, then it is convergent.
Definition 164: Monotone sequence
Let \((a_n)\) be a real sequence. We say that:
\((a_n)\) is increasing if \[ a_n \leq a_{n+1} \,, \quad \forall \, n \geq N \,. \]
\((a_n)\) is decreasing if \[ a_n \geq a_{n+1} \,, \quad \forall \, n \geq N \,. \]
\((a_n)\) is monotone if it is either increasing or decreasing.
Example 165
Solution. We have \[ a_{n+1} = \frac{n}{n+1} > \frac{n-1}{n} = a_n \,, \] where the inequality holds because \[\begin{align*} \frac{n}{n+1} > \frac{n-1}{n} \quad & \iff \quad n^2 > (n-1)(n+1) \quad \\ & \iff \quad n^2 > n^2 - 1 \\ & \iff \quad 0 > - 1 \end{align*}\]
Example 166
Solution. We have \[ a_n = \frac1n > \frac{1}{n+1} = a_{n+1} \,, \] concluding.
The main result about monotone sequences is the Monotone Convergence Theorem.
Theorem 167: Monotone Convergence Theorem
Proof
Assume \((a_n)\) is bounded and monotone. Since \((a_n)\) is bounded, the set \[ A:=\{ a_n \, \colon \,n \in \mathbb{N}\} \subseteq \mathbb{R} \] is bounded below and above. By the Axiom of Completeness of \(\mathbb{R}\) there exist \(i,s \in \mathbb{R}\) such that \[ i = \inf A \,, \quad s = \sup A \,. \]
We have two cases:
\((a_n)\) is increasing: We are going to prove that \[ \lim_{n \to \infty} a_n = s \,. \] Equivalently, we need to prove that \[ \forall \, \varepsilon>0 \,, \, \exists \, N \in \mathbb{N}\, \text{ s.t. } \, \forall \, n \geq N \,, \,\, |a_n - s| < \varepsilon\,. \tag{A.39}\] Let \(\varepsilon> 0\). Since \(s\) is the smallest upper bound for \(A\), this means \[ s - \varepsilon \] is not an upper bound. Therefore there exists \(N \in \mathbb{N}\) such that \[ s - \varepsilon< a_N \,. \tag{A.40}\] Let \(n \geq N\). Since \(a_n\) is increasing, we have \[ a_N \leq a_n \,, \quad \forall \, n \geq N \,. \tag{A.41}\] Moreover \(s\) is the supremum of \(A\), so that \[ a_n \leq s < s + \varepsilon\,, \quad \forall \, n \in \mathbb{N}\,. \tag{A.42}\] Putting together estimates (A.40)-(A.41)-(A.42) we get \[ s - \varepsilon< a_N \leq a_n \leq s < s + \varepsilon\,, \quad \forall \, n \geq N \,. \] The above implies \[ s - \varepsilon< a_n < s + \varepsilon\,, \quad \forall \, n \geq N \,, \] which is equivalent to (A.39).
\((a_n)\) is decreasing: With a similar proof, one can show that \[ \lim_{n \to \infty} a_n = i \,. \] This is left as an exercise.
A.9.1 Example: Euler’s Number
As an application of the Monotone Convergence Theorem we can give a formal definition for the Euler’s Number \[ e = 2.71828182845904523536 \dots \]
Theorem 168
We have that:
- \((a_n)\) is monotone increasing,
- \((a_n)\) is bounded.
In particular \((a_n)\) is convergent.
Proof
Part 2. We have to prove that \((a_n)\) is bounded, that is, that there exists \(M > 0\) such that \[ |a_n| \leq M \,, \quad \forall \, n \in \mathbb{N}\,. \] To this end, introduce the sequence \((b_n)\) by setting \[ b_n := \left( 1 + \frac1n \right)^{n+1} \,. \] The sequence \((b_n)\) is decreasing.
To prove \((b_n)\) is decreasing, we need to show that \[ b_{n-1} \geq b_n \,, \quad \forall \, n \in \mathbb{N}\,. \] By definition of \(b_n\), the above reads \[ \left( 1 + \frac{1}{n-1} \right)^{n} \geq \left( 1 + \frac{1}{n} \right)^{n+1} \,, \quad \forall \, n \in \mathbb{N}\,. \] Summing the terms inside the brackets, the above is equivalent to \[ \left( \frac{n}{n-1} \right)^{n} \geq \left( \frac{n+1}{n} \right)^{n} \left( \frac{n+1}{n} \right) \,. \] Multiplying by \((n/(n+1))^n\) we get \[ \left( \frac{n^2}{n^2 - 1} \right)^{n} \geq \left( \frac{n+1}{n} \right) \,. \] The above is equivalent to \[ \left( 1 + \frac{1}{n^2 - 1} \right)^{n} \geq \left( 1 + \frac{1}{n} \right) \,. \tag{A.44}\] Therefore \((b_n)\) is decreasing if and only if (A.44) holds for all \(n \in \mathbb{N}\). By choosing \[ x = \frac{1}{n^2 - 1} \] in Bernoulli’s inequality, we obtain \[\begin{align*} \left( 1 + \frac{1}{n^2 - 1} \right)^{n} & \geq 1 + n \left( \frac{1}{n^2 - 1} \right) \\ & = 1 + \frac{n}{n^2 - 1} \\ & \geq 1 + \frac1n \,, \end{align*}\] where in the last inequality we used that \[ \frac{n}{n^2 - 1} > \frac1n \,, \] which holds, being equivalent to \(n^2 > n^2 - 1\). We have therefore proven (A.44), and hence \((b_n)\) is decreasing.
We now observe that For all \(n \in \mathbb{N}\) \[\begin{align*} b_n & = \left( 1 + \frac1n \right)^{n+1} \\ & = \left( 1 + \frac1n \right)^n \left( 1 + \frac1n \right) \\ & = a_n \left( 1 + \frac1n \right) \\ & > a_n \,. \end{align*}\] Since \((a_n)\) is increasing and \((b_n)\) is decreasing, in particular \[ a_n \geq a_1 \,, \quad b_n \leq b_1 \,. \] Therefore \[ a_1 \leq a_n < b_n \leq b_1 \,, \quad \forall \, n \in \mathbb{N}\,. \] We compute \[ a_1 = 2 \,, \quad b_1 = 4 \,, \] from which we get \[ 2 \leq a_n \leq 4 \,, \quad \forall \, n \in \mathbb{N}\,. \] Therefore \[ |a_n| \leq 4 \,, \quad \forall \, n \in \mathbb{N}\,, \] showing that \((a_n)\) is bounded.
Part 3. The sequence \((a_n)\) is increasing and bounded above. Therefore \((a_n)\) is convergent by the Monotone Convergence Theorem 167.
Thanks to Theorem 168 we can define the Euler’s Number \(e\).
Definition 169: Euler’s Number
Setting \(n=1000\) in the formula for \((a_n)\), we get an approximation of \(e\): \[ e \approx a_{1000} = 2.7169 \,. \]
A.10 Some important limits
In this section we investigate limits of some sequences to which the Limit Tests do not apply.
Theorem 170
Proof
Step 2. Assume \(0< x < 1\). In this case \[ \frac{1}{x} > 1 \,. \] Therefore \[ \lim_{n \to \infty} \, \sqrt[n]{ 1/x } = 1 \,. \] by Step 1. Therefore \[ \sqrt[n]{x} = \frac{1}{\sqrt[n]{ 1/x }} \longrightarrow \frac{1}{1} = 1\,, \] by the Algebra of Limits.
Theorem 171
Proof
Step 1. We prove that \[ \sin(a_n) \to 0 \,. \] By elementary trigonometry we have \[ 0 \leq |\sin (x)| = \sin |x| \leq |x| \,, \quad \forall \, x \in \left[ -\frac{\pi}{2}, \frac{\pi}{2} \right] \,. \] Therefore, since (A.45) holds, we can substitute \(x=a_n\) in the above inequality to get \[ 0 \leq |\sin (a_n)| \leq |a_n| \,, \quad \forall \, n \geq \mathbb{N}\,. \] Since \(a_n \to 0\), we also have \(|a_n|\to 0\). Therefore \(|\sin (a_n)| \to 0\) by the Squeeze Theorem. This immediately implies \(\sin(a_n) \to 0\).
Step 2. We prove that \[ \cos(a_n) \to 1 \,. \] Inverting the relation \[ \cos^2(x) + \sin^2 (x) = 1 \,, \] we obtain \[ \cos(x) = \pm \sqrt{ 1 - \sin^2 (x) } \,. \] We have that \(\cos(x) \geq 0\) for \(-\pi/2 \leq x \leq \pi/2\). Thus \[ \cos(x) = \sqrt{ 1 - \sin^2 (x) } \,, \quad \forall \, x \in \left[ -\frac{\pi}{2}, \frac{\pi}{2} \right] \,. \] Since (A.45) holds, we can set \(x=a_n\) in the above inequality and obtain \[ \cos(a_n) = \sqrt{ 1 - \sin^2 (a_n) } \,, \quad \forall \, n \geq N \,. \] By Step 1 we know that \(\sin(a_n) \to 0\). Therefore, by the Algebra of Limits, \[ 1 - \sin^2 (a_n) \longrightarrow 1 - 0 \cdot 0 = 1 \,. \] Using Theorem 148 we have \[ \cos(a_n) = \sqrt{1 - \sin^2 (a_n)} \longrightarrow \sqrt{1} = 1 \,, \] concluding the proof.
Theorem 172
Proof
Warning
Theorem 173
Proof
Step 2. We have \[ \frac{1 - \cos(a_n)}{a_n} = a_n \cdot \frac{1 - \cos(a_n)}{(a_n)^2} \longrightarrow 0 \cdot \frac{1}{2} = 0 \,, \] using Step 1 and the Algebra of Limits.
Example 174
Solution. This is because \[ n \sin \left( \frac{1}{n} \right) = \frac{ \sin \left( \dfrac{1}{n} \right) }{ \dfrac{1}{n} } \longrightarrow 1 \,, \] by Theorem 172 with \(a_n = 1/n\).
Example 175
Solution. Indeed, \[ n^2 \left( 1 - \cos \left( \frac{1}{n} \right)\right) = \dfrac{1 - \cos \left( \dfrac{1}{n} \right)}{\dfrac{1}{n^2}} \longrightarrow \frac12 \,, \] by applying Theorem 173 with \(a_n = 1/n\).
Example 176
Solution. Using (A.48)-(A.47) and the Algebra of Limits \[\begin{align*} \frac{n \left( 1- \cos \left( \dfrac{1}{n} \right) \right) }{ \sin \left( \dfrac{1}{n} \right) } & = \frac{n^2 \left( 1- \cos \left( \dfrac{1}{n} \right) \right) }{ n \sin \left( \dfrac{1}{n} \right) } \\ & \longrightarrow \frac{1/2}{1} = \frac12 \,. \end{align*}\]
Example 177
Solution. We have \[ \cos \left( \frac{2}{n} \right) \longrightarrow 1 \,, \] by Theorem 171 applied with \(a_n = 2/n\). Moreover \[ \frac{\sin \left( \dfrac{2}{n} \right)}{\dfrac{2}{n}} \longrightarrow 1 \,, \] by Theorem 172 applied with \(a_n = 2/n\). Therefore \[\begin{align*} n \cos \left( \frac{2}{n} \right) \sin \left( \frac{2}{n} \right) & = 2 \cdot \cos \left( \frac{2}{n} \right) \cdot \frac{\sin \left( \dfrac{2}{n} \right)}{\dfrac{2}{n}} \\ & \longrightarrow 2 \cdot 1 \cdot 1 = 2 \,, \end{align*}\] where we used the Algebra of Limits.
Example 178
Solution. Note that \[\begin{align*} \frac{n^2+1}{n+1} \sin \left( \dfrac{1}{n} \right) & = \left( \frac{1+\dfrac{1}{n^2}}{1+ \dfrac{1}{n}} \right) \cdot \left( n \sin \left( \dfrac{1}{n} \right) \right) \\ & \longrightarrow \frac{1 + 0}{1 + 0} \cdot 1 = 1\,, \end{align*}\] where we used (A.47) and the Algebra of Limits.